Difference between revisions of "2007 USAMO Problems/Problem 1"
(→Solution 2) |
m (cleaned up typos and typesetting errors in Solution 1) |
||
Line 7: | Line 7: | ||
== Solution == | == Solution == | ||
=== Solution 1 === | === Solution 1 === | ||
− | Create an integer sequence <math>\displaystyle b_1, b_2, \ldots</math> such that for <math>\displaystyle k \ge 1</math>, <math>b_k = \displaystyle \left(\sum_{i=1}^{k}\right) / k</math>. Consider what happens when <math>b_k \le k</math>. For <math>\displaystyle k + 1</math>, we have <math>b_{k+1} = \displaystyle \left(\sum_{i=1}^{k+1}\right) / (k+1) = \left(\displaystyle \sum_{i=1}^{k} + a_{k+1}\right) / (k+1) = \frac{b_k \times k + a_{k+1}}{k+1}</math>. <math>\displaystyle b_k</math> is a permissible value of <math> a_{k+1} \displaystyle </math> since <math>b_k \le k = (k+1)-1</math>: if we [[substitute]] <math>\displaystyle b_k</math> for <math> a_{k+1} \displaystyle </math>, we get <math>b_{k+1} = \frac{b_k (k+1)}{k+1} = b_k</math>. <math>\displaystyle b_k</math> is the unique value for <math> a_{k+1} \displaystyle </math>. We can repeat this argument for <math>\displaystyle b_k = b_{k+1} = b_{k+2} \ | + | Create an integer sequence <math>\displaystyle b_1, b_2, \ldots</math> such that for <math>\displaystyle k \ge 1</math>, <math>b_k = \displaystyle \left(\sum_{i=1}^{k} a_i\right) / k</math>. Consider what happens when <math>b_k \le k</math>. For <math>\displaystyle k + 1</math>, we have <math>b_{k+1} = \displaystyle \left(\sum_{i=1}^{k+1}a_i \right) / (k+1) = \left(\displaystyle \sum_{i=1}^{k} a_i + a_{k+1}\right) / (k+1) = \frac{b_k \times k + a_{k+1}}{k+1}</math>. Note that <math>\displaystyle b_k</math> is a permissible value of <math> a_{k+1} \displaystyle </math> since <math>b_k \le k = (k+1)-1</math>: if we [[substitute]] <math>\displaystyle b_k</math> for <math> a_{k+1} \displaystyle </math>, we get <math>b_{k+1} = \frac{b_k (k+1)}{k+1} = b_k</math>. Therefore <math>\displaystyle b_k</math> is the unique value for <math> a_{k+1} \displaystyle </math>. We can repeat this argument for <math>\displaystyle b_k = b_{k+1} = b_{k+2} = \cdots</math>. As we substituted the <math>\displaystyle b_k</math>s for the <math>\displaystyle a_k</math>s, the <math>\displaystyle a_k</math>s become constant. |
− | Now we must show that <math>\displaystyle b_k | + | Now we must show that eventually <math>\displaystyle b_k \le k</math>. Suppose that <math>\displaystyle b_k > k</math> for all <math>k</math>. By definition, <math> \displaystyle \left(\sum_{i=1}^{k} a_i\right) / k = b_k > k</math>, so <math>\displaystyle \sum_{i=1}^{k} a_i > k^2</math>. We also have that for <math>i>1</math>, each <math>a_i \le i-1</math> so that <math>\displaystyle k^2 < \sum_{i=1}^{k} a_i \le n + 1 + 2 + \cdots + (k-1) = n + \frac{k^2 - k}2</math>. So <math>k^2 < n +\frac{k^2 - k}2 \Longrightarrow \frac{k^2 + k}2 < n</math>. But <math>\displaystyle n</math> is constant while <math>\displaystyle k</math> is increasing, so eventually we will have a contradiction and <math>b_k \le k</math>. Therefore, the sequence of <math>\displaystyle a_i</math>s will become constant. |
=== Solution 2 === | === Solution 2 === | ||
− | Define <math>\displaystyle s_k=a_1+a_2+ | + | Define <math>\displaystyle s_k=a_1+a_2+\cdots+a_k</math>. If we have that <math>\displaystyle s_k=j*k</math> where <math>j</math> is an integer such that <math>0\le j\le k</math>, then we can take <math>\displaystyle a_{k+1}=j</math>, and we are done since <math>\displaystyle s_{k+1}=a_{k+1}+s_k=j+j*k=j*(k+1)</math>. |
We always know that <math>s_k</math> is a [[multiple]] of <math>k</math> by definition of the sequence. So we know an integer ,<math>j</math>, exists. But we also need the [[inequality]] (<math>0\le j\le k</math>) for <math>j</math> to be able to add <math>j</math> as the next term. We need that, for some <math>k</math>, <math>s_k=k*j\le k^2</math> | We always know that <math>s_k</math> is a [[multiple]] of <math>k</math> by definition of the sequence. So we know an integer ,<math>j</math>, exists. But we also need the [[inequality]] (<math>0\le j\le k</math>) for <math>j</math> to be able to add <math>j</math> as the next term. We need that, for some <math>k</math>, <math>s_k=k*j\le k^2</math> | ||
− | However, we also know that <math>s_k=a_1+a_2+ | + | However, we also know that <math>s_k=a_1+a_2+\cdots+a_k\le n+1+2+\cdots+k-1=n+\frac{(k-1)(k)}{2}</math>. |
So we note that for sufficiently large <math>k</math>, we have that <math>n+\frac{(k-1)(k)}{2}\le k^2\iff n\le \frac{k^2+k}{2}</math>, hence <math>s_k\le k^2</math>. Then we are done, since we can take that suitable <math>j</math> and keep adding it. | So we note that for sufficiently large <math>k</math>, we have that <math>n+\frac{(k-1)(k)}{2}\le k^2\iff n\le \frac{k^2+k}{2}</math>, hence <math>s_k\le k^2</math>. Then we are done, since we can take that suitable <math>j</math> and keep adding it. |
Revision as of 19:41, 30 April 2007
Problem
Let be a positive integer. Define a sequence by setting and, for each , letting be the unique integer in the range for which is divisible by . For instance, when the obtained sequence is . Prove that for any the sequence eventually becomes constant.
Solution
Solution 1
Create an integer sequence such that for , . Consider what happens when . For , we have . Note that is a permissible value of since : if we substitute for , we get . Therefore is the unique value for . We can repeat this argument for . As we substituted the s for the s, the s become constant.
Now we must show that eventually . Suppose that for all . By definition, , so . We also have that for , each so that . So . But is constant while is increasing, so eventually we will have a contradiction and . Therefore, the sequence of s will become constant.
Solution 2
Define . If we have that where is an integer such that , then we can take , and we are done since .
We always know that is a multiple of by definition of the sequence. So we know an integer ,, exists. But we also need the inequality () for to be able to add as the next term. We need that, for some ,
However, we also know that .
So we note that for sufficiently large , we have that , hence . Then we are done, since we can take that suitable and keep adding it.
See also
2007 USAMO (Problems • Resources) | ||
Preceded by First question |
Followed by Problem 2 | |
1 • 2 • 3 • 4 • 5 • 6 | ||
All USAMO Problems and Solutions |